Linear Algebra infinite number of solutions

In summary, the conversation discusses solving a system of equations with more variables than equations using matrix operations. The responder gives tips on how to manipulate the matrix to get a leading 1 in each row and zeros below it. They also explain how to use the augmented matrix to find parameters for the variables. The conversation also touches on another problem and how to solve it using the same techniques.
  • #1
_Bd_
109
0

Homework Statement


just started this semester, any help will be appreciated.

4x + 12y - 7z - 20w = 22
3x + 9y - 5z - 28w = 30

solve the system

Homework Equations



well since I have more variables than equations I know there's an infinite number of solutions therefore I am going to have to use parameters later on. (BTW can someone give me tips on how to do parameters?)



The Attempt at a Solution



I have my matrix as follows
4 12 -7 -20 22
3 9 -5 -28 30

multiplying row_2 by -1 and adding it to row_1 I get
1 3 2 8 -8
3 9 -5 -28 30
multiplying row_1 by -3 and adding it to row_2 yields
1 3 2 8 -8
0 0 -11 -52 -54

then what? if I divide by 11 I start getting fractions, and the answer in the back doesn't have any fractions.

AFAIK you have to make the first one in each row 1 and have a zero below it. (condition on row_1 is met)
 
Physics news on Phys.org
  • #2
_Bd_ said:

Homework Statement


just started this semester, any help will be appreciated.

4x + 12y - 7z - 20w = 22
3x + 9y - 5z - 28w = 30

solve the system

Homework Equations



well since I have more variables than equations I know there's an infinite number of solutions therefore I am going to have to use parameters later on. (BTW can someone give me tips on how to do parameters?)



The Attempt at a Solution



I have my matrix as follows
4 12 -7 -20 22
3 9 -5 -28 30

multiplying row_2 by -1 and adding it to row_1 I get
This is wasted effort. The idea is to make the leading entry of a row 1, and the corresponding entry of a row below 0. What you should do instead is to add (-3) times row 1 to 4 times row 2. That will make the 2nd row
0 0 1 -52 54

After that, use the leading 1 in row 2 to eliminate the entry above it.
_Bd_ said:
1 3 2 8 -8
3 9 -5 -28 30
multiplying row_1 by -3 and adding it to row_2 yields
1 3 2 8 -8
0 0 -11 -52 -54

then what? if I divide by 11 I start getting fractions, and the answer in the back doesn't have any fractions.

AFAIK you have to make the first one in each row 1 and have a zero below it. (condition on row_1 is met)
 
  • #3
what do you mean? If i have 0 0 1 -52 54 I'd have the top row as the original 4 12 -7 -20 22. . .I can't do anything to the 4 (unless I make fractions) to get it into a 1

cause that's what I did actually, I started by making the leading coefficient on row_1 1, then I planned on using that to cancel out the zeros below it. . .and so on, I don't really get what you're doing, I am 'ma try it out on paper tho.
 
  • #4
You missed what I said, which was to use the leading 1 in the 2nd row to eliminate the -7 in the row above it.

Then replace row 1 by 1/4 of itself. I don't end up with any fractions.
 
  • #5
oh ok ok, I got it! so I did that and came up with:
1 3 0 96 100
0 0 1 -52 54

so then. . .now my problem is to do the parameters. . .

I can say that

x_4 = 1/52 {54-x_3}
and
x_4 = 1/96 {100-x_1-3x_2}

. . .altho I think I am tripping out and going in a tangent. . .how should I do to make parameters (like. . .any tips overall?)

and again thank you.

. . .I was thinking if I solve for x_1 on the first equation and replug back and plug back again I can get x_4 and x_1 in both in terms of x_2 and x_3. . .I guess that would be the answer?


===== on another problem =====

3x + 3y + 12z = 6
x + y + 4z = 2
2x + 5y + 20z = 10
-x + 2y + 8z = 4

I started the way I usually do (from top to bottom, dividing the top row by 3 and using that leading 1 to cancel the 1 on the second row and so forth)

but when I noticed I was getting the exact same lanes. . .

1 1 4 2
0 1 4 2
0 1 4 2

I erased everything so I can't copy all but I remember one of the lines just got ocmpletely elimiated into zeros. . .
 
Last edited:
  • #6
_Bd_ said:
oh ok ok, I got it! so I did that and came up with:
1 3 0 96 100
0 0 1 -52 54

so then. . .now my problem is to do the parameters. . .

I can say that

x_4 = 1/52 {54-x_3}
and
x_4 = 1/96 {100-x_1-3x_2}
I think that's what I got, but I don't have my work with me, so can't say for sure.
Edit: You're off in one sign in the top row. For your augmented matrix, you should have
1 3 0 -96 100
0 0 1 -52 54

I have edited what I wrote earlier to account for this difference.
Anyway, read off the two lines you have, keeping in mind that this is an augmented matrix.

x1 + 3x2 - 96x4 = 100
x3 -52x4 = 54

Now solve for the first variable in each equation.
x1 = -3x2 + 96x4 + 100
x3 = 52x4 + 54

Since there are no equations for x2 and x4, they are arbitrary.
_Bd_ said:
. . .altho I think I am tripping out and going in a tangent. . .how should I do to make parameters (like. . .any tips overall?)

and again thank you.

. . .I was thinking if I solve for x_1 on the first equation and replug back and plug back again I can get x_4 and x_1 in both in terms of x_2 and x_3. . .I guess that would be the answer?


===== on another problem =====

3x + 3y + 12z = 6
x + y + 4z = 2
2x + 5y + 20z = 10
-x + 2y + 8z = 4

I started the way I usually do (from top to bottom, dividing the top row by 3 and using that leading 1 to cancel the 1 on the second row and so forth)

but when I noticed I was getting the exact same lanes. . .

1 1 4 2
0 1 4 2
0 1 4 2

I erased everything so I can't copy all but I remember one of the lines just got ocmpletely elimiated into zeros. . .
 
Last edited:
  • #7
_bd_ said:
but when I noticed I was getting the exact same lanes. . .

1 1 4 2
0 1 4 2
0 1 4 2

I erased everything so I can't copy all but I remember one of the lines just got ocmpletely elimiated into zeros. . .
That's not a problem. It just means you will have two free variables, hence an infinite no. of solutions.
 
  • #8
I have now to do modeling, I have to find the equation that passes trough given points:

we know its a cubic equation with horizontal tangents at (1,-2) and (-1,2)
since the points seem "simetric" I assumed the graph passes trough the origin (0,0)

so I get the following

a_0 + a_1 (x_1) + a_2 (x_1)^2 + a_3 (x_1)^3 = y_1
a_0 + a_1 (x_2) + a_2 (x_2)^2 + a_3 (x_2)^3 = y_1
a_0 + a_1 (x_3) + a_2 (x_3)^2 + a_3 (x_3)^3 = y_1

when I do the matrix I can't instantly notice that a_0 is zero (therefore there is no constant)

once I do the matrix I have
1 1 1 -2
-1 1 -1 2
0 0 0 0

so my third equation cancels. . .I have 3 variables and only 2 equations... AFAIK you can't solve it (unless you parametize)
I continued and tried to reduce and ended up with
a_1 a_2 a_3
1 0 1 -2
0 1 0 0
which tells me that a_0 =0 and a_1 =0

the answer at the back shows p(x) = -3x + x^3
 

1. What is an infinite number of solutions in linear algebra?

In linear algebra, an infinite number of solutions refers to a system of equations that has an infinite number of possible solutions. This means that there are multiple combinations of values for the variables that satisfy all the equations in the system.

2. How can a system of equations have an infinite number of solutions?

A system of equations can have an infinite number of solutions when the equations are dependent on each other, meaning that one or more equations can be obtained by adding or subtracting multiples of the other equations. This creates an infinite number of equivalent equations and therefore an infinite number of solutions.

3. Can a system of equations have both a finite and an infinite number of solutions?

No, a system of equations can only have either a finite or an infinite number of solutions. If the equations are dependent and have an infinite number of solutions, then they cannot also have a unique solution. On the other hand, if the equations are independent and have a unique solution, then they cannot also have an infinite number of solutions.

4. How can we determine if a system of equations has an infinite number of solutions?

To determine if a system of equations has an infinite number of solutions, we can use the concept of rank. If the rank of the coefficient matrix is equal to the rank of the augmented matrix, then the system has an infinite number of solutions. This means that there are more variables than equations, allowing for multiple combinations of values to satisfy the equations.

5. What is the significance of a system having an infinite number of solutions?

A system of equations having an infinite number of solutions means that the equations are not enough to uniquely determine the values of the variables. This can occur in real-world scenarios where there are more unknowns than equations, making it impossible to find a unique solution. In these cases, we can only find a set of solutions that satisfy the equations, rather than a single solution.

Similar threads

  • Calculus and Beyond Homework Help
Replies
8
Views
622
  • Calculus and Beyond Homework Help
Replies
5
Views
290
  • Calculus and Beyond Homework Help
Replies
13
Views
1K
  • Calculus and Beyond Homework Help
Replies
7
Views
1K
  • Calculus and Beyond Homework Help
Replies
2
Views
989
  • Calculus and Beyond Homework Help
Replies
5
Views
2K
  • Calculus and Beyond Homework Help
Replies
9
Views
1K
  • Calculus and Beyond Homework Help
Replies
6
Views
810
  • Calculus and Beyond Homework Help
Replies
6
Views
3K
  • Calculus and Beyond Homework Help
Replies
5
Views
2K
Back
Top